What Are the Values of c for Continuity at x=5?

Click For Summary
To determine the values of c for continuity of the function at x=5, the two pieces of the function must be equal at that point. This involves setting the left-hand limit, f(5) from the first piece, equal to the right-hand limit, f(5) from the second piece, resulting in the equation 25 - c^2 = 5c - 11. Solving this equation will yield the two values of c that ensure continuity. It is essential to check that the function's value at x=5 matches these limits for continuity to hold. The correct approach involves evaluating limits from both sides and ensuring they are equal.
togame
Messages
18
Reaction score
0

Homework Statement


f(x) = x^2 - c^2 \mbox{ if } x < 5
f(x) = cx+11 \mbox{ if } x \geq 5

Find the two values of c for which the function would be continuous.


Homework Equations





The Attempt at a Solution


I set these two equations equal to each other, plug in the value 5 since that is the point at which these equations would meet, then solve for c? I'm not sure if I'm missing a step in the algebra or something else, but I seem to be unable to get the correct answer.
 
Last edited:
Physics news on Phys.org
togame said:

Homework Statement


f(x) = x^2 - c^2 \mbox{ if } x < 5
f(x) = cx+11 \mbox{ if } x \geq 5

Find the two values of c for which the function would be continuous.

Homework Equations



The Attempt at a Solution


I set these two equations equal to each other, plug in the value 5 since that is the point at which these equations would meet, then solve for c? I'm not sure if I'm missing a step in the algebra or something else, but I seem to be unable to get the correct answer.
Technically: You should find the limit of f(x) as x approaches 5 from the left, and then from the right and set the limits equal to each other, solving for c. Also, make sure that f(5) is the same as those limits.

Of course, when you do that, you do get (5)^2-c^2=5c-11\,.
 
Question: A clock's minute hand has length 4 and its hour hand has length 3. What is the distance between the tips at the moment when it is increasing most rapidly?(Putnam Exam Question) Answer: Making assumption that both the hands moves at constant angular velocities, the answer is ## \sqrt{7} .## But don't you think this assumption is somewhat doubtful and wrong?

Similar threads

  • · Replies 27 ·
Replies
27
Views
2K
  • · Replies 11 ·
Replies
11
Views
2K
Replies
22
Views
2K
  • · Replies 20 ·
Replies
20
Views
3K
  • · Replies 4 ·
Replies
4
Views
1K
Replies
7
Views
1K
  • · Replies 3 ·
Replies
3
Views
1K
  • · Replies 2 ·
Replies
2
Views
1K
Replies
1
Views
2K
Replies
8
Views
2K